This paper was converted on www.awesomepapers.org from LaTeX by an anonymous user.
Want to know more? Visit the Converter page.

On variants of Chowla’s conjecture

Krishnarjun Krishnamoorthy [email protected] Beijing Institute of Mathemtical Sciences and Applications (BIMSA), No. 544, Hefangkou Village, Huaibei Town, Huairou District, Beijing. To my parents
Abstract.

We study the shifted convolution sums associated to completely multiplicative functions taking values in {±1}\{\pm 1\} and give combinatorical proofs of two recent results in the direction of Chowla’s conjecture. We also determine the corresponding ”spectrum”.

Key words and phrases:
Chowla’s conjecture, Completely multiplicative functions, Shifted convolution sums, Spectrum
2020 Mathematics Subject Classification:
Primary : 11N37, 11P32, Secondary : 11N35, 11T06

1. Introduction

1.1. Completely multiplicative functions

A function f:f:\mathbb{N}\to\mathbb{C} is called completely multiplicative if for every m,nm,n\in\mathbb{N}, f(mn)=f(m)f(n)f(mn)=f(m)f(n). It follows that completely multiplicative functions are completely described by their values at primes. Let λ\lambda denote the Liouville function, defined to equal 1-1 at the primes and extended to natural numbers completely multiplicatively. The study of various averages related to the Liouville function is an important part of analytic number theory and is intimately tied with many outstanding conjectures such as the Riemann hypothesis etc. One such question regards the dd-point correlations of the Liouville function, which is the focus of this paper. In particular we have the following conjecture of Chowla [3].

Conjecture 1 (Chowla).

Let λ\lambda denote the Liouville function and let H={h1,,hd}H=\{h_{1},\ldots,h_{d}\} be a non-empty set of non-negative integers, then

(1.1) limx1xnxλ(n+h1)λ(n+hd)=0.\lim_{x\to\infty}\frac{1}{x}\sum_{n\leqslant x}\lambda(n+h_{1})\ldots\lambda(n+h_{d})=0.

There are many versions of this conjecture available in literature, a few replacing the Liouville function with the Möbius function (see [15] for more information and the inter-dependencies of various versions of Conjecture 1). In all the above variants, the conjecture encodes the expectation that the prime decomposition of an integer nn should be independent of that of n+hn+h for any fixed h1h\geqslant 1 (or other suitable polynomial shifts). Thus the values of Liouville function at these integers maybe treated as “independent events” and hence their average should vanish in the long run (that is they should not be correlated). Extending this philosophy to dd-shifts gives Conjecture 1.

1.2. Main results

In this paper we give combinatorical proofs of two recent results regarding the shifted convolution averages of completely multiplicative functions taking values in {±1}\{\pm 1\}, particularly along the lines of Conjecture 1. We begin with some notation. Let PP denote a subset of primes. Associated to PP, we define the completely multiplicative function λP\lambda_{P} as

(1.2) λP(p):={1 if pP1 otherwise.\lambda_{P}(p):=\begin{cases}-1&\mbox{ if }p\in P\\ 1&\mbox{ otherwise.}\end{cases}

Equivalently, if ΩP(n)\Omega_{P}(n) denotes the completely additive function defined on the primes as

(1.3) ΩP(p):={1 if pP0 otherwise.\Omega_{P}(p):=\begin{cases}1&\mbox{ if }p\in P\\ 0&\mbox{ otherwise.}\end{cases}

then λP(n)=(1)ΩP(n)\lambda_{P}(n)=(-1)^{\Omega_{P}(n)}. Every completely multiplicative function taking values in {±1}\{\pm 1\} is of the form λP\lambda_{P} for some PP. In fact this association to a completely multiplicative function taking values in {±1}\{\pm 1\}, the set of primes where it takes the value 1-1 is a group isomorphism where the subsets of primes are considered as a group under symmetric difference and the completely multiplicative functions are considered under pointwise multiplication.

Recall that a subset of natural numbers is called a small set if its sum of reciprocals converges and called a large set otherwise. For H={h1,,hd}H=\{h_{1},\ldots,h_{d}\} we define

(1.4) ΛPH(n):=λP(n+h1)λP(n+hd).\Lambda_{P}^{H}(n):=\lambda_{P}(n+h_{1})\ldots\lambda_{P}(n+h_{d}).

Define H^:={hihj|hi,hjH,hihj}\hat{H}:=\{h_{i}-h_{j}\ |\ h_{i},h_{j}\in H,h_{i}\neq h_{j}\}. Call a prime exceptional if it divides an element of H^\hat{H} and non-exceptional otherwise. We now have the first main theorem of this paper.

Theorem 1.

Let H={h1,,hd}H=\{h_{1},\ldots,h_{d}\} be a fixed subset of \mathbb{N}. Let PP be a small set of primes. Then, for every prime pp, there exists constants ηpH\eta_{p}^{H} (depending on HH) such that

(1.5) κPH:=limx1xnxΛPH(n)=pP(12ηpH).\kappa_{P}^{H}:=\lim_{x\to\infty}\frac{1}{x}\sum_{n\leqslant x}\Lambda_{P}^{H}(n)=\prod_{p\in P}\left(1-2\eta_{p}^{H}\right).

Moreover, for every non-exceptional prime, ηpH=dp+1\eta_{p}^{H}=\frac{d}{p+1}.

The above theorem can be deduced from the results in [10], but our proofs are different and comparitively elementary. What is interesting however is that the right hand side of (1.5) resembles an Euler product even when the corresponding arithmetic function namely ΛPH(n)\Lambda_{P}^{H}(n) fails to be even multiplicative. Moreover, it is not even clear that the associated Dirichlet series (namely n=1ΛPH(n)ns\sum_{n=1}^{\infty}\Lambda_{P}^{H}(n)n^{-s}) continues beyond the half plane (s)>1\Re(s)>1; a property much needed for the application of Tauberian arguments. The multiplicative nature of the right hand side of (1.5) in turn, conforms with the principle that the “global” density should be a product of the “local” densities.

Theorem 1 still leaves open the question of large subsets of primes. In this case, we expect the limit on the left hand side of (1.5) to vanish, thus making Theorem 1 true in this case as well (if we interpret the infinite product on the right of (1.5) as zero, similar to the infinite product on the right hand side of (1.8) below). In particular, this would imply Conjecture 1. While such a generalization of Theorem 1 is out of reach, we are however able to prove the following.

Theorem 2.

If there exists ϕH\phi\neq H\subset\mathbb{N} such that |κPH|=1|\kappa_{P}^{H}|=1, then P=ϕP=\phi. More concretely, if λP:{±1}\lambda_{P}:\mathbb{N}\to\{\pm 1\} is a non constant completely multiplicative function, then for any H={h1,,hd}H=\{h_{1},\ldots,h_{d}\},

(1.6) lim infx1xnxλP(n+h1)λP(n+hd)<1,\liminf_{x\to\infty}\frac{1}{x}\sum_{n\leqslant x}\lambda_{P}(n+h_{1})\ldots\lambda_{P}(n+h_{d})<1,

and

(1.7) lim supx1xnxλP(n+h1)λP(n+hd)>1\limsup_{x\to\infty}\frac{1}{x}\sum_{n\leqslant x}\lambda_{P}(n+h_{1})\ldots\lambda_{P}(n+h_{d})>-1

Theorem 2 follows from a recent work of Teravainen [20], which is much more general. As mentioned before the purpose of the present article is to give an elementary combinatorical proof.

Given a family of multiplicative functions taking values inside the unit circle, their “spectrum” was studied in [6]. Analogously, we define the spectrum as ΓH\Gamma_{H} to be the topological closure of the set of all values κPH\kappa_{P}^{H} as PP runs through all small subsets of primes for a fixed HH. The precise evaluation of ηpH\eta_{p}^{H} below allows us to describe the spectrum ΓH\Gamma_{H} below in §5.

1.3. Brief overview of the proof

We give a brief overview of the proof of Theorems 1 and Theorem 2 leaving the details for the following sections. Define 𝒩PH:={n|ΛPH(n)=1}\mathcal{N}_{P}^{H}:=\left\{n\in\mathbb{N}\ |\ \Lambda_{P}^{H}(n)=-1\right\} and set ηPH=δ(𝒩PH)\eta_{P}^{H}=\delta\left(\mathcal{N}_{P}^{H}\right) (we shall show later on that in the cases that interest us, this natural density exists). The key idea in the proofs of Theorems 1 and 2 is the description of some structure in certain interesting families of subsets of \mathbb{N}.

For Theorem 1, the key ingredient is Lemma 2, which describes how 𝒩PH\mathcal{N}_{P}^{H} behaves if we change PP and HH. This allows us to prove Theorem 1 for finite sets of primes and to let the set of primes grow “adding one prime at a time”. This leads to a limiting process which converges only if the original set PP is a small set. The relevant inequality is (3.9). As a consequence of the proof we describe how to calculate ηPH\eta_{P}^{H} (see Example 1). From [10], ηpH\eta_{p}^{H} is connected to the number of roots of the polynomial (x+h1)(x+hd)(x+h_{1})\ldots(x+h_{d}) modulo powers of pp. We provide a slightly different description.

The proof of Theorem 2 involves a combinatorical argument which is rather independent in itself. The idea is to show that the collection of counterexamples satisfy some additional structure and symmetries (see Lemma 4). Using this we may reduce the the proof to the two-point correlation case, which is settled due to the work of Matomaki and Radziwill. The main arithmetic input, along with some handy lemmas, is Theorem 4 below (see also Remark 1).

1.4. Known results

Before we proceed with the proof, we mention some important results available in literature. The only case where Conjecture 1 is known is when d=1d=1, in which case the statement is classically equivalent to the prime number theorem. Using the fact that λP\lambda_{P} is completely multiplicative, we may derive (for (s)>1\Re(s)>1)

n=1λP(n)ns=ζ(s)pP(1ps1+ps)\sum_{n=1}^{\infty}\frac{\lambda_{P}(n)}{n^{s}}=\zeta(s)\prod_{p\in P}\left(\frac{1-p^{-s}}{1+p^{-s}}\right)

where ζ(s)\zeta(s) is the Riemann zeta function. Combining the works of Wintner [21] and Wirsing [22] leads to the following theorem (see also [1, Theorem 2]).

Theorem 3 (Wintner-Wirsing).

For any subset PP of the primes, we have

(1.8) limx1xnxλP(n)=pP(12p+1).\lim_{x\to\infty}\frac{1}{x}\sum_{n\leqslant x}\lambda_{P}(n)=\prod_{p\in P}\left(1-\frac{2}{p+1}\right).

An important aspect of the above theorem is the existence of the limit on the left hand side of (1.8).

Apart from the case of d=1d=1, it was not even known whether the limit in (1.1) exists. Even if it did exist, it was unknown whether there is sufficient cancellations to ensure that the limit is better than the trivial limit (that is ±1\pm 1). This second question was settled only recently, due to the important work of Matomäki and Radziwiłł [11], where they studied the relation between “short” and “long” averages of arithmetic functions. We state, as a theorem, a corollary of the main result of [11].

Theorem 4 (Matomäki - Radziwiłł).

Let h1h\geqslant 1 be an integer, then there exists a positive constant δ(P,h)\delta(P,h) depending on PP and hh such that

1x|nxλP(n)λP(n+h)|1δ(P,h)\frac{1}{x}\left|\sum_{n\leqslant x}\lambda_{P}(n)\lambda_{P}(n+h)\right|\leqslant 1-\delta(P,h)

for all large enough x>1x>1 and for all PP non-empty. In other words,

(1.9) lim supx1x|nxλP(n)λP(n+h)|<1\limsup_{x\to\infty}\frac{1}{x}\left|\sum_{n\leqslant x}\lambda_{P}(n)\lambda_{P}(n+h)\right|<1

for any h1h\geqslant 1, and PϕP\neq\phi.

Most of the recent progress, particularly in the last decade or so regarding the Chowla’s conjecture (or more generally the Hardy-Littlewood-Chowla conjectures) have been from the perspective of ergodic theory. Following [11], there has been some progress towards Conjecture 1 particularly focusing on averaged [12, 13] and logarithmic [16, 18] versions. For conditional results assuming the existence of Siegel zeros, see [19]. For more information111The author wishes to thank Stelios Sachpazis for directing him to these references., see also [2, 9, 17, 4, 5, 14, 7].

Notation

  1. (1)

    Let ={1,2,}\mathbb{N}=\{1,2,\ldots\} denote the natural numbers and 0\mathbb{N}_{0} denote the non-negative integers {0}\mathbb{N}\cup\{0\}.

  2. (2)

    The symbol δ\delta will denote natural density (whenever it exists), δ+\delta^{+} and δ\delta^{-} will denote the upper and lower natural densities respectively.

  3. (3)

    We shall denote the empty set as ϕ\phi. The empty sum is defined to be zero and the empty product is defined to be one.

  4. (4)

    Given any two subsets A,B0A,B\subseteq\mathbb{N}_{0}, we define their “sum” as A+B:={a+b|aA,bB}A+B:=\{a+b\ |\ a\in A,b\in B\}, and their “product” as AB:={ab|aA,bB}A\cdot B:=\{ab\ |\ a\in A,b\in B\}. In particular, if A={a}A=\{a\}, we also write a+Ba+B and aBaB in place of A+BA+B and ABA\cdot B respectively.

  5. (5)

    The symbol PP will usually denote subsets of prime numbers unless otherwise stated. The symbol HH will usually refer to a finite subset of 0\mathbb{N}_{0}.

2. Groundwork

Given a sequence {an}(1,1)\{a_{n}\}\subset(-1,1), we define the infinite product n=1(1an)\prod_{n=1}^{\infty}(1-a_{n}) as limTn=1T(1an)\lim_{T\to\infty}\prod_{n=1}^{T}(1-a_{n}). The product is considered to be convergent if and only if an\sum a_{n} is convergent, in which case the limit above exists. If an\sum a_{n} diverges to \infty, then the product is defined to be 0. Given any two sets A,BA,B, their symmetric difference ABA\triangle B is defined as (AB)(BA)(A\setminus B)\cup(B\setminus A). In particular, ABA\triangle B contains elements of ABA\cup B belonging to exactly one of AA or BB. Furthermore, by induction, the symmetric difference of a finite number of sets {Ai}\{A_{i}\} is precisely the collection of those elements belonging to exactly an odd number of the AiA_{i}’s. Suppose that PP is a non-empty subset of primes, we let 𝒮P\mathcal{S}_{P} denote the set of PP smooth integers. That is

𝒮P:={n| all the prime factors of n are in P}.\mathcal{S}_{P}:=\{n\in\mathbb{N}\ |\ \mbox{ all the prime factors of }n\mbox{ are in }P\}.

It follows from our convention that 1𝒮P1\in\mathcal{S}_{P} for every PP and 𝒮ϕ={1}\mathcal{S}_{\phi}=\{1\}.

Lemma 1.

For any two subsets P1,P2P_{1},P_{2} of the primes, and for any nn\in\mathbb{N},

λP1P2(n)=λP1(n)λP2(n).\lambda_{P_{1}\triangle P_{2}}(n)=\lambda_{P_{1}}(n)\lambda_{P_{2}}(n).
Proof.

Suppose first that P1P_{1} and P2P_{2} are disjoint. Factorize nn as n=n1n2n3n=n_{1}n_{2}n_{3} where n1n_{1} is the largest divisor of nn in 𝒮P1\mathcal{S}_{P_{1}}, n2n_{2} that in 𝒮P2\mathcal{S}_{P_{2}} and n3n_{3} defined as n/(n1n2)n/(n_{1}n_{2}). It follows that λP1(n)=λP1(n1),λP2(n)=λP2(n2)\lambda_{P_{1}}(n)=\lambda_{P_{1}}(n_{1}),\lambda_{P_{2}}(n)=\lambda_{P_{2}}(n_{2}) and λP1P2(n)=λP1P2(n1n2)\lambda_{P_{1}\cup P_{2}}(n)=\lambda_{P_{1}\cup P_{2}}(n_{1}n_{2}). Thus it suffices to show that

(2.1) λP1(n1)λP2(n2)=λP1P2(n1n2).\lambda_{P_{1}}(n_{1})\lambda_{P_{2}}(n_{2})=\lambda_{P_{1}\cup P_{2}}(n_{1}n_{2}).

But observe that λP1(n1)=λ(n1)\lambda_{P_{1}}(n_{1})=\lambda(n_{1}) and likewise for the other two terms. Thus the lemma follows from the multiplicativity of the Liouville function.

For the general case write nn as the product n=n1n2n3n4n=n_{1}n_{2}n_{3}n_{4} where n1,n2,n3n_{1},n_{2},n_{3} are respectively the largest divisors of nn in 𝒮P1P2,𝒮P1P2,𝒮P2P1\mathcal{S}_{P_{1}\setminus P_{2}},\mathcal{S}_{P_{1}\cap P_{2}},\mathcal{S}_{P_{2}\setminus P_{1}}, and n4n_{4} is defined as n/(n1n2n3)n/(n_{1}n_{2}n_{3}). Then on repeatedly applying (2.1), we have

λP1(n)λP2(n)=λP1P2(n)λP1P22(n)λP2P1(n)=λP1P2(n)λP2P1(n)=λP1P2(n).\lambda_{P_{1}}(n)\lambda_{P_{2}}(n)=\lambda_{P_{1}\setminus P_{2}}(n)\lambda^{2}_{P_{1}\cap P_{2}}(n)\lambda_{P_{2}\setminus P_{1}}(n)=\lambda_{P_{1}\setminus P_{2}}(n)\lambda_{P_{2}\setminus P_{1}}(n)=\lambda_{P_{1}\triangle P_{2}}(n).

From our conventions, 𝒩Pϕ=ϕ\mathcal{N}_{P}^{\phi}=\phi for any PP.

Lemma 2.

For any two subsets P1,P2P_{1},P_{2} of the primes and two finite subsets H1,H2H_{1},H_{2} of 0\mathbb{N}_{0},

𝒩P1P2H1H2=𝒩P1H1𝒩P1H2𝒩P2H1𝒩P2H2.\mathcal{N}_{P_{1}\triangle P_{2}}^{H_{1}\triangle H_{2}}=\mathcal{N}_{P_{1}}^{H_{1}}\triangle\mathcal{N}_{P_{1}}^{H_{2}}\triangle\mathcal{N}_{P_{2}}^{H_{1}}\triangle\mathcal{N}_{P_{2}}^{H_{2}}.
Proof.

The proof is given in two parts. First we show that 𝒩P1P2H=𝒩P1H𝒩P2H\mathcal{N}_{P_{1}\triangle P_{2}}^{H}=\mathcal{N}_{P_{1}}^{H}\triangle\mathcal{N}_{P_{2}}^{H} for any HH. If H=ϕH=\phi, the claim follows from definition, so suppose not. Let m:=(n+h1)(n+hd)m:=(n+h_{1})\ldots(n+h_{d}), where we have chosen H={h1,,hd}H=\{h_{1},\ldots,h_{d}\}. From Lemma 1 and complete multiplicativity,

ΛP1P2H(n)=λP1P2H(m)=λP1P2H(m)λP2P1H(m)=ΛP1P2H(n)ΛP2P1H(n).\Lambda_{P_{1}\triangle P_{2}}^{H}(n)=\lambda_{P_{1}\triangle P_{2}}^{H}(m)=\lambda_{P_{1}\setminus P_{2}}^{H}(m)\lambda_{P_{2}\setminus P_{1}}^{H}(m)=\Lambda_{P_{1}\setminus P_{2}}^{H}(n)\Lambda_{P_{2}\setminus P_{1}}^{H}(n).

Thus ΛP1P2H(n)=1\Lambda_{P_{1}\triangle P_{2}}^{H}(n)=-1 if and only if ΛP1P2H(n)ΛP2P1H(n)\Lambda_{P_{1}\setminus P_{2}}^{H}(n)\neq\Lambda_{P_{2}\setminus P_{1}}^{H}(n). But, we observe that ΛP1H(n)=ΛP1P2H(n)ΛP1P2H(n)\Lambda_{P_{1}}^{H}(n)=\Lambda_{P_{1}\setminus P_{2}}^{H}(n)\Lambda_{P_{1}\cap P_{2}}^{H}(n) and ΛP2H(n)=ΛP2P1H(n)ΛP1P2H(n)\Lambda_{P_{2}}^{H}(n)=\Lambda_{P_{2}\setminus P_{1}}^{H}(n)\Lambda_{P_{1}\cap P_{2}}^{H}(n). Thus, ΛP1P2H(n)ΛP2P1H(n)\Lambda_{P_{1}\setminus P_{2}}^{H}(n)\neq\Lambda_{P_{2}\setminus P_{1}}^{H}(n) if and only if ΛP1H(n)ΛP2H(n)\Lambda_{P_{1}}^{H}(n)\neq\Lambda_{P_{2}}^{H}(n). To conclude, ΛP1P2H(n)=1\Lambda_{P_{1}\triangle P_{2}}^{H}(n)=-1 if and only if ΛP1H(n)ΛP2H(n)=1\Lambda_{P_{1}}^{H}(n)\Lambda_{P_{2}}^{H}(n)=-1, that is exactly one of them is 1-1. The latter condition is satisfied precisely on 𝒩P1H𝒩P2H\mathcal{N}_{P_{1}}^{H}\triangle\mathcal{N}_{P_{2}}^{H}.

Now we show that 𝒩PH1H2=𝒩PH1𝒩PH2\mathcal{N}_{P}^{H_{1}\triangle H_{2}}=\mathcal{N}_{P}^{H_{1}}\triangle\mathcal{N}_{P}^{H_{2}}. If either H1,H2H_{1},H_{2} is the empty set, then the claim follows from definition, so we suppose not. Let H1={h1,,hd1}H_{1}=\{h_{1},\ldots,h_{d_{1}}\} and H2={k1,,kd2}H_{2}=\{k_{1},\ldots,k_{d_{2}}\}. Observe that n𝒩PH1n\in\mathcal{N}_{P}^{H_{1}} if and only if ΩP((n+h1)(n+hd1))=i=1d1ΩP(n+hi)\Omega_{P}((n+h_{1})\ldots(n+h_{d_{1}}))=\sum_{i=1}^{d_{1}}\Omega_{P}(n+h_{i}) is odd, and similarly for H2H_{2}. Let H1H2={l1,,ld}H_{1}\triangle H_{2}=\{l_{1},\ldots,l_{d}\}.

If n𝒩PH1H2n\in\mathcal{N}_{P}^{H_{1}\triangle H_{2}}, then suppose without loss of generality that ΩP(n+li)\Omega_{P}(n+l_{i}) is odd for 1ie1\leqslant i\leqslant e (say) (with liH1H2l_{i}\in H_{1}\triangle H_{2}) and even otherwise. In particular exactly one of |{l1,,le}(H1H2)||\{l_{1},\ldots,l_{e}\}\cap(H_{1}\setminus H_{2})| or |{l1,,le}(H2H1)||\{l_{1},\ldots,l_{e}\}\cap(H_{2}\setminus H_{1})| is odd. Suppose without loss of generality that the former holds. Let {r1,,re}H1H2\{r_{1},\ldots,r_{e^{\prime}}\}\subseteq H_{1}\cap H_{2} denote the complete set of integers such that ΩP(n+ri)\Omega_{P}(n+r_{i}) is odd. If ee^{\prime} is odd, then n𝒩PH2𝒩PH1n\in\mathcal{N}_{P}^{H_{2}}\setminus\mathcal{N}_{P}^{H_{1}}, and if ee^{\prime} is even, then n𝒩PH1𝒩PH2n\in\mathcal{N}_{P}^{H_{1}}\setminus\mathcal{N}_{P}^{H_{2}}. In any case n𝒩PH1𝒩PH2n\in\mathcal{N}_{P}^{H_{1}}\triangle\mathcal{N}_{P}^{H_{2}}. Therefore 𝒩PH1H2𝒩PH1𝒩PH2\mathcal{N}_{P}^{H_{1}\triangle H_{2}}\subseteq\mathcal{N}_{P}^{H_{1}}\triangle\mathcal{N}_{P}^{H_{2}}.

Now we prove the reverse inclusion. Suppose without loss of generality that n𝒩PH1𝒩PH2n\in\mathcal{N}_{P}^{H_{1}}\setminus\mathcal{N}_{P}^{H_{2}}. Let us suppose (with notation as above) that ee^{\prime} is even. The other case may be treated similarly. Choose lil_{i} as above. Then from our choices, it follows that |{l1,,le}(H1H2)||\{l_{1},\ldots,l_{e}\}\cap(H_{1}\setminus H_{2})| is odd and |{l1,,le}(H2H1)||\{l_{1},\ldots,l_{e}\}\cap(H_{2}\setminus H_{1})| is even. In particular, ee is odd and hence n𝒩PH1H2n\in\mathcal{N}_{P}^{H_{1}\triangle H_{2}}. This completes the proof. ∎

For r,sr,s\in\mathbb{N}, define the arithmetic progression

C(r,s):={n|nrmods}.C(r,s):=\left\{n\in\mathbb{N}\ |\ n\equiv-r\mod s\right\}.

It follows from the Chinese remainder theorem that if a,ba,b are co-prime integers, then for any r,sr,s, there exists a unique tt such that

(2.2) C(r,a)C(s,b)=C(t,ab).C(r,a)\bigcap C(s,b)=C(t,ab).

Furthermore,

(2.3) δ(C(r,s))=1s\delta(C(r,s))=\frac{1}{s}

for any r,sr,s\in\mathbb{N}. Combining (2.2) and (2.3), we get

δ(C(r,a)C(s,b))=δ(C(r,a))δ(C(s,b))\delta\left(C(r,a)\bigcap C(s,b)\right)=\delta\left(C(r,a)\right)\delta\left(C(s,b)\right)

whenever a,ba,b are co-prime. Define 𝒜P\mathcal{A}_{P} to be the collection all finite unions of the sets

{ϕ}{C(r,n)|r,n𝒮P}.\{\phi\}\cup\left\{C(r,n)\ |\ r\in\mathbb{N},\ n\in\mathcal{S}_{P}\right\}.
Lemma 3.

For any non-empty subset PP of the primes, the following statements are true.

  1. (1)

    𝒜P\mathcal{A}_{P} is closed under intersection.

  2. (2)

    𝒜P\mathcal{A}_{P} is closed under complements.

  3. (3)

    Every element of 𝒜P\mathcal{A}_{P} can be written as a finite disjoint union of sets of the form {C(r,n)}\{C(r,n)\} for a fixed n𝒮Pn\in\mathcal{S}_{P}.

  4. (4)

    The natural density exists for every set A𝒜PA\in\mathcal{A}_{P}.

Proof.
  1. (1)

    It is enough to prove this assertion for sets of the form C(r,n)C(r,n) for n𝒮Pn\in\mathcal{S}_{P}. If C(r1,n1)C(r2,n2)=ϕC(r_{1},n_{1})\cap C(r_{2},n_{2})=\phi, we are done. Otherwise, C(r1,n1)C(r2,n2)C(r_{1},n_{1})\cap C(r_{2},n_{2}) is a finite union of sets of the form C(r,n3)C(r,n_{3}) where n3n_{3} is least common multiple of n1n_{1} and n2n_{2}. Therefore n3𝒮Pn_{3}\in\mathcal{S}_{P} and we are done.

  2. (2)

    From above, it is sufficient to show that Cc(r,n)𝒜PC^{c}(r,n)\in\mathcal{A}_{P}. Clearly Cc(r,n)=sr,1snC(s,n)𝒜PC^{c}(r,n)=\cup_{s\neq r,1\leqslant s\leqslant n}C(s,n)\in\mathcal{A}_{P}.

  3. (3)

    Suppose n1,,nk𝒮Pn_{1},\ldots,n_{k}\in\mathcal{S}_{P}. Denote by NN, the least common multiple of {n1,,nk}\{n_{1},\ldots,n_{k}\} and set mi=N/nim_{i}=N/n_{i} for every 1ik1\leqslant i\leqslant k. We have,

    i=1kC(ri,ni)=i=1kj=1miC(ri+jni,N).\bigcup_{i=1}^{k}C(r_{i},n_{i})=\bigcup_{i=1}^{k}\bigcup_{j=1}^{m_{i}}C(r_{i}+jn_{i},N).

    The right hand side maybe written as a disjoint union by avoiding repetitions if there are any.

  4. (4)

    The natural density clearly exists for every set of the form C(r,n)C(r,n) and the claim follows from above. This completes the proof.

Proposition 1.

Suppose that P1,P2P_{1},P_{2} are non-empty mutually disjoint subsets of primes. Then δ(AB)=δ(A)δ(B)\delta(A\cap B)=\delta(A)\delta(B) for any A𝒜P1A\in\mathcal{A}_{P_{1}} and B𝒜P2B\in\mathcal{A}_{P_{2}}.

Proof.

Suppose AA and BB are given as above. Then from Lemma 3, there exists N𝒮P1N\in\mathcal{S}_{P_{1}} and M𝒮P2M\in\mathcal{S}_{P_{2}} such that

A=i=1k1C(ri,N)\displaystyle A=\bigcup_{i=1}^{k_{1}}C(r_{i},N) and B=j=1k2C(sj,M),\displaystyle B=\bigcup_{j=1}^{k_{2}}C(s_{j},M),

where each of the above is a disjoint union. Therefore from the Chinese remainder theorem (see (2.2)), there exists distinct (modulo NMNM) tlt_{l}’s such that

AB=l=1k1k2C(tl,NM).A\cap B=\bigcup_{l=1}^{k_{1}k_{2}}C(t_{l},NM).

From here, we may deduce that δ(AB)=k1k2NM=δ(A)δ(B)\delta(A\cap B)=\frac{k_{1}k_{2}}{NM}=\delta(A)\delta(B). ∎

3. Proof of Theorem 1

3.1. Proof of Theorem 1 when PP is finite:

We simplify notation and write 𝒩ph\mathcal{N}_{p}^{h} for 𝒩{p}{h}\mathcal{N}_{\{p\}}^{\{h\}}, Ωp\Omega_{p} for Ω{p}\Omega_{\{p\}} etc.

Proposition 2.

Given a finite subset PP of the primes and H0H\subseteq\mathbb{N}_{0} and an integer rr, there exists subsets XPH(r),YPH(r)X_{P}^{H}(r),Y_{P}^{H}(r) in 𝒜P\mathcal{A}_{P} such that

XPH(r)𝒩PHYPH(r)X_{P}^{H}(r)\subseteq\mathcal{N}_{P}^{H}\subseteq Y_{P}^{H}(r)

and

limrδ(XPH(r))=limrδ(YPH(r)).\lim_{r\to\infty}\delta(X_{P}^{H}(r))=\lim_{r\to\infty}\delta(Y_{P}^{H}(r)).
Proof.

The proof is by double induction, first on |H||H| and then on |P||P|. Let us first prove the statement for a fixed prime pp and when H={h}H=\{h\}. Then 𝒩ph\mathcal{N}_{p}^{h} contains precisely those integers nn such that an odd power of pp properly divides n+hn+h. In particular,

(3.1) 𝒩ph=r=1(C(h,p2r1)C(h,p2r)).\mathcal{N}_{p}^{h}=\bigcup_{r=1}^{\infty}\left(C(h,p^{2r-1})\setminus C(h,p^{2r})\right).

Since the above union is a disjoint union, we have the inequalities,

(3.2) t=1r(C(h,p2t1)C(h,p2t))𝒩pht=1r(C(h,p2t1)C(h,p2t))C(h,p2r1)\bigcup_{t=1}^{r}\left(C(h,p^{2t-1})\setminus C(h,p^{2t})\right)\subseteq\mathcal{N}_{p}^{h}\subseteq\bigcup_{t=1}^{r}\left(C(h,p^{2t-1})\setminus C(h,p^{2t})\right)\bigcup C(h,p^{2r-1})

for any rr. Call the sets on the left hand side and right hand side of (3.2) as Xph(r)X_{p}^{h}(r) and Yph(r)Y_{p}^{h}(r) respectively. We observe that δ(Xph(r))δ(Yph(r))δ(Xph(r))+p2r+1\delta(X_{p}^{h}(r))\leqslant\delta(Y_{p}^{h}(r))\leqslant\delta(X_{p}^{h}(r))+p^{-2r+1}. Letting rr\to\infty, we see that δ(𝒩ph)\delta(\mathcal{N}_{p}^{h}) exists and equals

(3.3) δ(𝒩ph)=r=1(1p2r11p2r)=1p+1.\delta\left(\mathcal{N}_{p}^{h}\right)=\sum_{r=1}^{\infty}\left(\frac{1}{p^{2r-1}}-\frac{1}{p^{2r}}\right)=\frac{1}{p+1}.

This completes the proof in this case.

Suppose that H={h1,,hd}H=\{h_{1},\ldots,h_{d}\} and consider 𝒩pH\mathcal{N}_{p}^{H}. Let φ:H/p\varphi:H\to\mathbb{Z}/p\mathbb{Z} denote the natural projection map. From Lemma 2,

(3.4) 𝒩pH=𝒩pφ1([0])𝒩pφ1([p1])=𝒩ph1𝒩phd\mathcal{N}_{p}^{H}=\mathcal{N}_{p}^{\varphi^{-1}([0])}\triangle\ldots\triangle\mathcal{N}_{p}^{\varphi^{-1}([p-1])}=\mathcal{N}_{p}^{h_{1}}\triangle\ldots\triangle\mathcal{N}_{p}^{h_{d}}

In particular, from (3.3), it follows that

(3.5) δ+(𝒩pH)dp+1.\delta^{+}\left(\mathcal{N}_{p}^{H}\right)\leqslant\frac{d}{p+1}.

But from (3.1), we may easily deduce that 𝒩phi𝒩phjϕ\mathcal{N}_{p}^{h_{i}}\cap\mathcal{N}_{p}^{h_{j}}\neq\phi only if p|(hihj)p|(h_{i}-h_{j}). Therefore we may rewrite the first equality in (3.4) as the disjoint union

(3.6) 𝒩pH=i=0p1𝒩pφ1([i]).\mathcal{N}_{p}^{H}=\bigcup_{i=0}^{p-1}\mathcal{N}_{p}^{\varphi^{-1}([i])}.

If φ\varphi is a non-constant function, then ηpH\eta_{p}^{H} exists by induction on |H||H| and we are done. In particular, we may suppose that H{0,1},{1,2}H\neq\{0,1\},\{1,2\} or {0,1,2}\{0,1,2\}.

Suppose that φ\varphi is a constant function, say φ(H)={[i1]}\varphi(H)=\{[i_{1}]\} for some 0i1p10\leqslant i_{1}\leqslant p-1. If ni1modpn\not\equiv-i_{1}\mod p, then p(n+h)p\nmid(n+h) for any hHh\in H and hence ΛpH(n)=1\Lambda_{p}^{H}(n)=1. Therefore, n𝒩pHn\in\mathcal{N}_{p}^{H} only if ni1modpn\equiv-i_{1}\mod p. Moreover, in this situation,

ΛpH(n)=λp(n+h1)λp(n+hd)=(1)dλp(np+h1p)λp(np+hdp).\Lambda_{p}^{H}(n)=\lambda_{p}(n+h_{1})\ldots\lambda_{p}(n+h_{d})=(-1)^{d}\lambda_{p}\left(\frac{n}{p}+\frac{h_{1}}{p}\right)\ldots\lambda_{p}\left(\frac{n}{p}+\frac{h_{d}}{p}\right).

Let H1:=1p(Hi1)H_{1}:=\frac{1}{p}(H-i_{1}). We observe that max{H1}<max{H}\max\{H_{1}\}<\max\{H\}. Rewriting the above,

(3.7) 𝒩pH={p𝒩pH1i1if 2|dp(𝒩pH1)ci1otherwise.\mathcal{N}_{p}^{H}=\begin{cases}p\mathcal{N}_{p}^{H_{1}}-i_{1}&\mbox{if }2|d\\ p\left(\mathcal{N}_{p}^{H_{1}}\right)^{c}-i_{1}&\mbox{otherwise.}\end{cases}

In either case, it is enough to prove the proposition for 𝒩pH1\mathcal{N}_{p}^{H_{1}}. Now, if we consider the natural projection map φ1:H1/p\varphi_{1}:H_{1}\to\mathbb{Z}/p\mathbb{Z}, we may argue as above. If φ1\varphi_{1} is a non-constant map, we are done. Otherwise we obtain (if necessary, after translation and scaling as above) H2:=1p(H1i2)H_{2}:=\frac{1}{p}(H_{1}-i_{2}), and the problem reduces to that of 𝒩pH2\mathcal{N}_{p}^{H_{2}}, with max{H2}<max{H1}\max\{H_{2}\}<\max\{H_{1}\}. Proceeding forward this process ultimately terminates because max{Hi}\max\{H_{i}\} is strictly decreasing. In particular, there is a stage rr where the projection map (say φr:Hr/p\varphi_{r}:H_{r}\to\mathbb{Z}/p\mathbb{Z}) is non-constant and we may then apply induction on |H||H|. This completes the proof when |P|=1|P|=1. We note in passing that for large enough primes pp, φ\varphi is an injection and hence from (3.3) and (3.6) we have ηpH=dp+1.\eta_{p}^{H}=\frac{d}{p+1}.

Now fix HH and suppose that P={p1,p2,,pk}P=\{p_{1},p_{2},\ldots,p_{k}\}. We shall prove the proposition holds for PP supposing the same for P:={p1,p2,,pk1}P^{\prime}:=\{p_{1},p_{2},\ldots,p_{k-1}\} and {pk}\{p_{k}\}. From Lemma 2,

𝒩PH=𝒩PH𝒩{pk}H.\mathcal{N}_{P}^{H}=\mathcal{N}_{P^{\prime}}^{H}\triangle\mathcal{N}_{\{p_{k}\}}^{H}.

For every rr, choose XPH(r),YPH(r),X{pk}H(r),Y{pk}H(r)X_{P^{\prime}}^{H}(r),Y_{P^{\prime}}^{H}(r),X_{\{p_{k}\}}^{H}(r),Y_{\{p_{k}\}}^{H}(r) from induction. Then, we have

(XPH(r)Y{pk}H(r))(X{pk}H(r)YPH(r))𝒩PH(YPH(r)X{pk}H(r))(Y{pk}H(r)XPH(r)).\left(X_{P^{\prime}}^{H}(r)\setminus Y_{\{p_{k}\}}^{H}(r)\right)\bigcup\left(X_{\{p_{k}\}}^{H}(r)\setminus Y_{P^{\prime}}^{H}(r)\right)\subseteq\mathcal{N}_{P}^{H}\subseteq\left(Y_{P^{\prime}}^{H}(r)\setminus X_{\{p_{k}\}}^{H}(r)\right)\bigcup\left(Y_{\{p_{k}\}}^{H}(r)\setminus X_{P^{\prime}}^{H}(r)\right).

From Lemma 3, both the left hand side and the right hand side above belong to 𝒜P\mathcal{A}_{P}. In particular the natural densities exist for each of those sets. Considering (upper) natural densities (and observing that XPH(r)YPH(r)X_{P^{\prime}}^{H}(r)\subseteq Y_{P^{\prime}}^{H}(r)) we get,

δ(XPH(r)Y{pk}H(r))+δ(X{pk}H(r)YPH(r))δ+(𝒩PH)δ(YPH(r)X{pk}H(r))+δ(Y{pk}H(r)XPH(r)).\delta\left(X_{P^{\prime}}^{H}(r)\setminus Y_{\{p_{k}\}}^{H}(r)\right)+\delta\left(X_{\{p_{k}\}}^{H}(r)\setminus Y_{P^{\prime}}^{H}(r)\right)\leqslant\delta^{+}(\mathcal{N}_{P}^{H})\\ \leqslant\delta\left(Y_{P^{\prime}}^{H}(r)\setminus X_{\{p_{k}\}}^{H}(r)\right)+\delta\left(Y_{\{p_{k}\}}^{H}(r)\setminus X_{P^{\prime}}^{H}(r)\right).

From Proposition 1, we get

δ(XP(r))(1δ(Y{pk}(r)))+δ(X{pk}(r))(1δ(YP(r)))δ+(𝒩P)δ(YP(r))(1δ(X{pk}(r)))+δ(Y{pk}(r))(1δ(XP(r))).\delta(X_{P^{\prime}}(r))(1-\delta(Y_{\{p_{k}\}}(r)))+\delta(X_{\{p_{k}\}}(r))(1-\delta(Y_{P^{\prime}}(r)))\leqslant\delta^{+}(\mathcal{N}_{P})\\ \leqslant\delta(Y_{P^{\prime}}(r))(1-\delta(X_{\{p_{k}\}}(r)))+\delta(Y_{\{p_{k}\}}(r))(1-\delta(X_{P^{\prime}}(r))).

Similarly, we may also consider lower natural densities. Letting rr\to\infty, we prove δ(𝒩PH)\delta(\mathcal{N}_{P}^{H}) exists and equals

(3.8) ηPH=ηPH(1ηpkH)+ηpkH(1ηPH).\eta_{P}^{H}=\eta_{P^{\prime}}^{H}(1-\eta_{p_{k}}^{H})+\eta_{p_{k}}^{H}(1-\eta_{P^{\prime}}^{H}).

Example 1.

Given a particular choice of HH and pp, it is possible to calculate ηpH\eta_{p}^{H} using the above arguments, particularly using (3.7). We briefly demonstrate this now for the choice of p=2,3p=2,3 and H={0,4,6}H=\{0,4,6\}. Observe that d=3d=3 is odd. On applying (3.6) and (3.7) repeatedly, we get

η2{0,4,6}=12(1η2{0,2,3})=12(1(η2{0,2}+η2{3}))=12(1(η2{0}+η2{1})2η2{3})=16.\eta_{2}^{\{0,4,6\}}=\frac{1}{2}\left(1-\eta_{2}^{\{0,2,3\}}\right)=\frac{1}{2}\left(1-\left(\eta_{2}^{\{0,2\}}+\eta_{2}^{\{3\}}\right)\right)=\frac{1}{2}\left(1-\frac{(\eta_{2}^{\{0\}}+\eta_{2}^{\{1\}})}{2}-\eta_{2}^{\{3\}}\right)=\frac{1}{6}.

where in the last step, we have used (3.3). Similarly,

η3{0,4,6}=η3{0,6}+η3{4}=13η3{0,2}+η3{4}=1324+14=512.\eta_{3}^{\{0,4,6\}}=\eta_{3}^{\{0,6\}}+\eta_{3}^{\{4\}}=\frac{1}{3}\eta_{3}^{\{0,2\}}+\eta_{3}^{\{4\}}=\frac{1}{3}\cdot\frac{2}{4}+\frac{1}{4}=\frac{5}{12}.

To complete the proof of Theorem 1 when PP is finite, we observe that

κPH=limx1xnxΛPH(n)=limx1x(|{nx|ΛPH(n)=1}||{nx|ΛPH(n)=1}|)=12ηPH.\kappa_{P}^{H}=\lim_{x\to\infty}\frac{1}{x}\sum_{n\leqslant x}\Lambda_{P}^{H}(n)=\\ \lim_{x\to\infty}\frac{1}{x}\left(\left|\left\{n\leqslant x\ |\ \Lambda_{P}^{H}(n)=1\right\}\right|-\left|\left\{n\leqslant x\ |\ \Lambda_{P}^{H}(n)=-1\right\}\right|\right)=1-2\eta_{P}^{H}.

Suppose that P={p1,p2,,pk}P=\{p_{1},p_{2},\ldots,p_{k}\} and P:={p1,p2,,pk1}P^{\prime}:=\{p_{1},p_{2},\ldots,p_{k-1}\} as above. From (3.8),

12ηPH=12(ηPH(1ηpkH)+ηpkH(1ηPH))=12ηPH+2ηPHηpkH2ηpkH+2ηpkHηPH=(12ηPH)(12ηpkH).1-2\eta_{P}^{H}=1-2\left(\eta_{P^{\prime}}^{H}(1-\eta_{p_{k}}^{H})+\eta_{p_{k}}^{H}(1-\eta_{P^{\prime}}^{H})\right)\\ =1-2\eta_{P^{\prime}}^{H}+2\eta_{P^{\prime}}^{H}\eta_{p_{k}}^{H}-2\eta_{p_{k}}^{H}+2\eta_{p_{k}}^{H}\eta_{P^{\prime}}^{H}=(1-2\eta_{P^{\prime}}^{H})(1-2\eta_{p_{k}}^{H}).

Now we may complete the proof via induction.

3.2. Proof of Theorem 1 when PP is a small set:

Suppose H={h1,,hd}H=\{h_{1},\ldots,h_{d}\} is fixed as above and that P={p1,p2,}P=\{p_{1},p_{2},\ldots\} is an infinite (small) set and let Pi={p1,p2,,pi}P_{i}=\{p_{1},p_{2},\ldots,p_{i}\}. We show that ηPH\eta_{P}^{H} exists and equals limiηPiH\lim_{i\to\infty}\eta_{P_{i}}^{H}. It will be convenient to set SP(x)=1xnxΛPH(n)S_{P}(x)=\frac{1}{x}\sum_{n\leqslant x}\Lambda_{P}^{H}(n). From Lemma 1, ΛPiH(n)ΛPH(n)\Lambda_{P_{i}}^{H}(n)\neq\Lambda_{P}^{H}(n) if and only if ΛPPiH(n)=1\Lambda_{P\setminus P_{i}}^{H}(n)=-1. Hence,

(3.9) lim supx|SPi(x)SP(x)|2δ+(𝒩PPiH)2pPPiηpH2dpPPi1p+1,\limsup_{x\to\infty}\left|S_{P_{i}}(x)-S_{P}(x)\right|\leqslant 2\delta^{+}\left(\mathcal{N}_{P\setminus P_{i}}^{H}\right)\leqslant 2\sum_{p\in P\setminus P_{i}}\eta_{p}^{H}\leqslant 2d\sum_{p\in P\setminus P_{i}}\frac{1}{p+1},

where the last inequality follows from (3.5). As PP is a small set, the right hand side goes to zero as ii\to\infty. This in particular shows ηPH\eta_{P}^{H} exists because the left hand side of (3.9) dominates

max{|lim supxSPi(x)lim infxSP(x)|,|lim supxSP(x)lim infxSPi(x)|}.\max\left\{\left|\limsup_{x\to\infty}S_{P_{i}}(x)-\liminf_{x\to\infty}S_{P}(x)\right|,\left|\limsup_{x\to\infty}S_{P}(x)-\liminf_{x\to\infty}S_{P_{i}}(x)\right|\right\}.

Moreover, this also implies that ηPH=limiηPiH\eta_{P}^{H}=\lim_{i\to\infty}\eta_{P_{i}}^{H} thereby completing the proof.

Example 2.

Interestingly if H={1,2}H=\{1,2\} and PP is a small set containing 33, the above calculations and proof give us that κPH=0\kappa_{P}^{H}=0. More generally, for any given prime odd pp, we may also choose H={1,,p+12}H=\{1,\ldots,\frac{p+1}{2}\} and any small set PP containing pp and get κPH=0\kappa_{P}^{H}=0.

4. Proof of Theorem 2

We denote the complement of 𝒩PH\mathcal{N}_{P}^{H} (in \mathbb{N}) as 𝒫PH\mathcal{P}_{P}^{H}. Recall that κPϕ=1\kappa_{P}^{\phi}=1.

Lemma 4.

If there exists sets P,H1,H2P,H_{1},H_{2} such that |κPH1|=|κPH2|=1|\kappa_{P}^{H_{1}}|=|\kappa_{P}^{H_{2}}|=1, then κPH1H2=κPH1κPH2\kappa_{P}^{H_{1}\triangle H_{2}}=\kappa_{P}^{H_{1}}\kappa_{P}^{H_{2}}.

Proof.

We have

κPH1=limx1xnxΛPH1H2(n)ΛPH1H2(n)=±1.\kappa_{P}^{H_{1}}=\lim_{x\to\infty}\frac{1}{x}\sum_{n\leqslant x}\Lambda_{P}^{H_{1}\setminus H_{2}}(n)\Lambda_{P}^{H_{1}\cap H_{2}}(n)=\pm 1.

In particular, this means that ΛPH1H2(n)=κPH1ΛPH1H2(n)\Lambda_{P}^{H_{1}\setminus H_{2}}(n)=\kappa_{P}^{H_{1}}\Lambda_{P}^{H_{1}\cap H_{2}}(n) on a set of natural density equal to 11. Reversing the roles of H1,H2H_{1},H_{2}, we obtain that ΛPH2H1(n)=κPH2ΛPH1H2(n)\Lambda_{P}^{H_{2}\setminus H_{1}}(n)=\kappa_{P}^{H_{2}}\Lambda_{P}^{H_{1}\cap H_{2}}(n) on a set of natural density equal to 11, and in particular ΛPH2H1(n)ΛPH1H2(n)=κPH1κPH2\Lambda_{P}^{H_{2}\setminus H_{1}}(n)\Lambda_{P}^{H_{1}\setminus H_{2}}(n)=\kappa_{P}^{H_{1}}\kappa_{P}^{H_{2}} on a set of natural density equal to 11. Thus we have shown that

κPH1H2=limx1xnxΛPH1H2(n)ΛPH2H1(n)=κPH1κPH2,\kappa_{P}^{H_{1}\triangle H_{2}}=\lim_{x\to\infty}\frac{1}{x}\sum_{n\leqslant x}\Lambda_{P}^{H_{1}\setminus H_{2}}(n)\Lambda_{P}^{H_{2}\setminus H_{1}}(n)=\kappa_{P}^{H_{1}}\kappa_{P}^{H_{2}},

completing the proof. ∎

We borrow the proof of the following lemma from combinatorics [8].

Lemma 5 (Wildon).

Suppose \mathcal{H} is a non-empty collection of finite subsets of 0\mathbb{N}_{0} which is closed under symmetric difference and translation, containing at least one non-empty set. Then \mathcal{H} contains a set of two elements.

By translation, we mean that if HH\in\mathcal{H} and aa\in\mathbb{Z} is such that H+a0H+a\subseteq\mathbb{N}_{0}, then H+aH+a\in\mathcal{H}.

Proof.

For a given prime power qq, let 𝔽q\mathbb{F}_{q} denote the field of qq elements. Consider the map φ:𝔽2(t)\varphi:\mathcal{H}\to\mathbb{F}_{2}(t) given by HhHthH\mapsto\sum_{h\in H}t^{h}. By convention φ(ϕ)=0\varphi(\phi)=0. We observe that φ(H1H2)=φ(H1)+φ(H2)\varphi(H_{1}\triangle H_{2})=\varphi(H_{1})+\varphi(H_{2}) and that φ(H+a)=taφ(H)\varphi(H+a)=t^{a}\varphi(H). These two observations together tell us that φ()\varphi(\mathcal{H}) is an ideal of 𝔽2(t)\mathbb{F}_{2}(t). Suppose this ideal is generated by f(t)f(t). Because \mathcal{H} is closed under translation as above, f(0)0f(0)\neq 0 (equivalently tf(t)t\nmid f(t)). Let 𝔽2r\mathbb{F}_{2^{r}} be the splitting field of ff. Allowing for multiple roots if needed, we see that there exists an integer mm such that f(t)|(t2r1+1)mf(t)|(t^{2^{r}-1}+1)^{m}, that is, there exists a polynomial q(t)q(t) such that q(t)f(t)=(t2r1+1)mq(t)f(t)=(t^{2^{r}-1}+1)^{m}. If nn is so that 2nm2^{n}\geqslant m, then

f(t)g(t)(t2r1+1)2nm=(t2r1+1)2n=t(2r1)2n+1.f(t)g(t)(t^{2^{r}-1}+1)^{2^{n}-m}=(t^{2^{r}-1}+1)^{2^{n}}=t^{(2^{r}-1)2^{n}}+1.

But the left hand side is an element of φ()\varphi(\mathcal{H}) and hence {1,(2r1)2n+1}\{1,(2^{r}-1)2^{n}+1\}\in\mathcal{H}. ∎

Proof of Theorem 2.

Suppose that PP is given such that |κPH0|=1|\kappa_{P}^{H_{0}}|=1 for some (non-empty) H0H_{0}. Let P\mathcal{H}_{P} denote the collection of all finite subsets HH of 0\mathbb{N}_{0} such that |κPH|=1|\kappa_{P}^{H}|=1. Clearly P\mathcal{H}_{P} is a non-empty collection and is closed under translation. From Lemma 4, P\mathcal{H}_{P} is closed under symmetric difference. Then by Lemma 5, P\mathcal{H}_{P} contains a set of order two. This however contradicts Theorem 4 unless P=ϕP=\phi. This completes the proof. ∎

Remark 1.

A family \mathcal{H} satisfying the conditions of Lemma 4 need not contain a singleton element. Thus, the above argument fails without the knowledge of Theorem 4. In particular, Theorem 3 is not sufficient to deduce Theorem 2.

Remark 2.

It is an desirable to prove an analogue of (1.9) in place of (1.6) and (1.7). However, the current proof does not seem to yield that strengthening. The challenge is to improve upon Lemma 4. More precisely, if two subsets of natural numbers have natural density equal to 11, then so does their intersection and this natural density is once again equal to 11. However, this is no longer true for upper or lower natural densities.

5. The spectrum of κPH\kappa_{P}^{H}

Let HϕH\neq\phi be given. Define αH:=infp{(12ηpH)}\alpha_{H}:=\inf_{p}\{(1-2\eta_{p}^{H})\} where the infimum runs over all the primes. Furthermore, we remark that there exists a prime pp such that αH=12ηpH\alpha_{H}=1-2\eta_{p}^{H}.

Proposition 3.

With notation as above, ΓH=[αH,1][0,1]\Gamma_{H}=[\alpha_{H},1]\cup[0,1].

Before we move on to the proof of Proposition 3, we require the following lemma. We give the proof for the sake of completeness.

Lemma 6.

For any given HH, α(0,1)\alpha\in(0,1) and a parameter XX, there exists a small set of primes PP, with each prime larger than XX such that κPH=α\kappa_{P}^{H}=\alpha.

Proof.

We shall choose XX large enough without loss of generality and suppose that every prime pXp\geqslant X is a non-exceptional prime. Choose a finite set P1P_{1} such that 1+α2>κP1H>α\frac{1+\alpha}{2}>\kappa_{P_{1}}^{H}>\alpha. Such a choice is possible because the sum of reciprocals of primes is divergent. For simplicity, suppose α1=α/κP1H\alpha_{1}=\alpha/\kappa_{P_{1}}^{H}. Choose P2P_{2} disjoint from P1P_{1} such that 1+α12>κP2H>α1\frac{1+\alpha_{1}}{2}>\kappa_{P_{2}}^{H}>\alpha_{1}. Define α2:=α1/κP2H\alpha_{2}:=\alpha_{1}/\kappa_{P_{2}}^{H}. In the next step, we choose P3P_{3} disjoint from P1P2P_{1}\cup P_{2} and so on. Constructing αi\alpha_{i}’s recursively as above, give for each ii

1+αi2>κPi+1H>αi.\frac{1+\alpha_{i}}{2}>\kappa_{P{i+1}}^{H}>\alpha_{i}.

Multiplying out by j=1iκPjH\prod_{j=1}^{i}\kappa_{P_{j}}^{H} gives us

(5.1) j=1iκPjH+α2>j=1i+1κPjH>α.\frac{\prod_{j=1}^{i}\kappa_{P_{j}}^{H}+\alpha}{2}>\prod_{j=1}^{i+1}\kappa_{P_{j}}^{H}>\alpha.

From the monotone convergence theorem, j=1i+1κPjH\prod_{j=1}^{i+1}\kappa_{P_{j}}^{H} is convergent. But then (5.1) forces the limit to be α\alpha. Now we choose PP to be i=1Pi\cup_{i=1}^{\infty}P_{i}. The fact that κPH=α\kappa_{P}^{H}=\alpha follows from Theorem 1 and the fact PP is a small set follows from the fact that α>0\alpha>0. ∎

Proof of Proposition 3.

Choose XX so that for primes p>Xp>X, ηpH>αH\eta_{p}^{H}>\alpha_{H}. If αH0\alpha_{H}\geqslant 0, then the proposition is clear from Lemma 6. Otherwise choose β[α,1]\beta\in[\alpha,1], we shall show that there exists a PP such that κPH=β\kappa_{P}^{H}=\beta. From Lemma 6, we may suppose that β(αH,0)\beta\in(\alpha_{H},0). Choose α:=β/αH\alpha:=\beta/\alpha_{H}, and choose PP a small set of non-exceptional primes, each larger than XX, such that κPH=12α\kappa_{P}^{H}=1-2\alpha. Let pp be such that αH=12ηpH\alpha_{H}=1-2\eta_{p}^{H}. Then from Theorem 1, the required small set of primes is P{p}P\cup\{p\}. Thus [αH,1][0,1]ΓH[\alpha_{H},1]\cup[0,1]\subseteq\Gamma_{H}.

Suppose αH>0\alpha_{H}>0. Suppose xΓHx\in\Gamma_{H}, it suffices to show that x0x\geqslant 0. This is clear from Theorem 1 since in this case (12ηpH)>0(1-2\eta_{p}^{H})>0 for every prime pp, so that ΓH[0,1]\Gamma_{H}\subset[0,1].

Suppose αH<0\alpha_{H}<0. For every small set PP, it then suffices to show that κPHαH\kappa_{P}^{H}\geqslant\alpha_{H}. Now observe that |κPH|1|\kappa_{P}^{H}|\leqslant 1 for every subset PP of the primes. Suppose 12ηpH=αH1-2\eta_{p}^{H}=\alpha_{H} for some fixed prime pp. For any small set PP, we have that κPHκP{p}HαHαH\kappa_{P}^{H}\geqslant\kappa_{P\setminus\{p\}}^{H}\alpha_{H}\geqslant\alpha_{H}. ∎

References

  • [1] Peter Borwein, Stephen K. K. Choi, and Michael Coons. Completely multiplicative functions taking values in {1,1}\{-1,1\}. Trans. Amer. Math. Soc., 362(12):6279–6291, 2010.
  • [2] Jake Chinis. Siegel zeros and sarnak’s conjecture, 2021.
  • [3] S. Chowla. The Riemann hypothesis and Hilbert’s tenth problem, volume Vol. 4 of Mathematics and its Applications. Gordon and Breach Science Publishers, New York-London-Paris, 1965.
  • [4] Jean-Marie De Koninck, László Germán, and Imre Kátai. On the convolution of the Liouville function under the existence of Siegel zeros. Lith. Math. J., 55(3):331–342, 2015.
  • [5] L. Germán and I. Kátai. On multiplicative functions on consecutive integers. Lith. Math. J., 50(1):43–53, 2010.
  • [6] Andrew Granville and K. Soundararajan. The spectrum of multiplicative functions. Ann. of Math. (2), 153(2):407–470, 2001.
  • [7] Harald Andrés Helfgott and Maksym Radziwiłł. Expansion, divisibility and parity, 2021.
  • [8] Mark Wildon (https://mathoverflow.net/users/7709/mark wildon). Collection closed under symmetric difference and translation. MathOverflow. URL:https://mathoverflow.net/q/484822 (version: 2024-12-26).
  • [9] Mikko Jaskari and Stelios Sachpazis. The chowla conjecture and landau-siegel zeroes, 2024.
  • [10] Oleksiy Klurman. Correlations of multiplicative functions and applications. Compos. Math., 153(8):1622–1657, 2017.
  • [11] Kaisa Matomäki and Maksym Radziwił ł. Multiplicative functions in short intervals. Ann. of Math. (2), 183(3):1015–1056, 2016.
  • [12] Kaisa Matomäki, Maksym Radziwiłł, and Terence Tao. An averaged form of Chowla’s conjecture. Algebra Number Theory, 9(9):2167–2196, 2015.
  • [13] Kaisa Matomäki, Maksym Radziwiłł, Terence Tao, Joni Teräväinen, and Tamar Ziegler. Higher uniformity of bounded multiplicative functions in short intervals on average. Ann. of Math. (2), 197(2):739–857, 2023.
  • [14] Cédric Pilatte. Improved bounds for the two-point logarithmic chowla conjecture, 2023.
  • [15] Olivier Ramaré. Chowla’s conjecture: from the Liouville function to the Moebius function. In Ergodic theory and dynamical systems in their interactions with arithmetics and combinatorics, volume 2213 of Lecture Notes in Math., pages 317–323. Springer, Cham, 2018.
  • [16] Terence Tao. The logarithmically averaged Chowla and Elliott conjectures for two-point correlations. Forum Math. Pi, 4:e8, 36, 2016.
  • [17] Terence Tao and Joni Teräväinen. Odd order cases of the logarithmically averaged Chowla conjecture. J. Théor. Nombres Bordeaux, 30(3):997–1015, 2018.
  • [18] Terence Tao and Joni Teräväinen. The structure of logarithmically averaged correlations of multiplicative functions, with applications to the Chowla and Elliott conjectures. Duke Math. J., 168(11):1977–2027, 2019.
  • [19] Terence Tao and Joni Teräväinen. The Hardy-Littlewood-Chowla conjecture in the presence of a Siegel zero. J. Lond. Math. Soc. (2), 106(4):3317–3378, 2022.
  • [20] Joni Teräväinen. On the Liouville function at polynomial arguments. Amer. J. Math., 146(4):1115–1167, 2024.
  • [21] Aurel Wintner. The Theory of Measure in Arithmetical Semi-Groups. publisher unknown, Baltimore, MD, 1944.
  • [22] E. Wirsing. Das asymptotische Verhalten von Summen über multiplikative Funktionen. II. Acta Math. Acad. Sci. Hungar., 18:411–467, 1967.